1 Introduction

Euler’s totient function \(\phi (n)\), which enumerates the number of positive integers which are co-prime to and less than or equal to n, is a classical arithmetical function. It is a well known fact that the number of solutions to the equation \(\phi (x) = m\) is finite for each \(m \in {\mathbb {N}}\) (\({\mathbb {N}}\) is the set of positive integers). It is natural, then, to ask the following questions:

  1. (i)

    For a given \(m \in {\mathbb {N}}\), what is the largest integer n such that \(\phi (n) \le m\)?

  2. (ii)

    What are the largest and the smallest integers satisfying \(\phi (x) = m\)?

We denote the set \(\{x : \phi (x) = m\}\) by \(\phi ^{-1}(m)\) and the image of \(\phi \) by V, i.e. \(V = \{\phi (m) : m \in {\mathbb {N}}\}\). The elements of V are called totients. For \(m \in V\), we define the following quantities with the above questions in mind:

$$\begin{aligned} N_1(m)&= \max \{x : \phi (x) \le m\}, \\ N_2(m)&= \max (\phi ^{-1}(m)), \\ N_3(m)&= \min (\phi ^{-1}(m)), \\ N_i&= \{N_i(m) : m \in V\} \ \text { for } i = 1,2,3. \end{aligned}$$

Note that \(N_2(m), N_3(m)\) are defined only on V, whereas \(N_1(m)\) can be defined on the whole of \({\mathbb {N}}\). But this does not contribute any new elements to the image \(N_1\) of \(N_1(m)\), since \(N_1(m) = N_1(m - 1)\) if \(m \notin V\). Hence, from here on, we study \(N_1(m)\) only for \(m \in V\). In 1986, Masser and Shiu [10] studied many properties of \(N_1\) and called its elements as ‘sparsely totient numbers’. They gave the following criteria to find examples of sparsely totient numbers.

PROPOSITION 1

[10] Let \((p_i)_{i=1}^{\infty }\) be the enumeration of the primes in ascending order. Suppose \(k\ge 2,\) \(d\ge 1\) and \(l\ge 0\) satisfy conditions \(d < p_{k+1}-1\) and \(d (p_{k+l}-1) < (d+1)(p_{k}-1).\) Then \(dp_1\cdots p_{k-1}p_{k+l}\) is a sparsely totient number.

They also found some nice patterns among sparsely totient numbers.

PROPOSITION 2

[10]

For \(n\in N_1\), let \(n'\) represent the smallest sparsely totient number greater than n. Then

  1. (i)

    \(\frac{n'}{n}\rightarrow 1 \text { as } n\rightarrow \infty \) and \(n\in N_1\).

  2. (ii)

    For a given prime p, \(\exists \ m(p)\in {\mathbb {N}}\) such that \(N_1(m)\equiv 0\pmod {p}\) \(\forall \) \(m \ge m(p)\).

This proposition suggests that the distribution of elements of \(N_1\) may be very sparse. To study the notion of sparseness of a subset of integers, we use properties like asymptotic density or Banach density. The asymptotic density gives the fraction of the number of elements of a set in \({\mathbb {N}}\) whereas Banach density gives an idea about how locally sparse or dense a set is. For example, the set \(\cup _{n \in {\mathbb {N}}}[10^n, 10^n + n]\) has asymptotic density zero but it has, in fact, a maximum Banach density of 1. The notion of Banach density will be defined in section 2. The first theorem in this paper measures the densities of sets \(V, N_i\), etc.

Theorem 3

  1. (i)

    The Banach density of V and \(N_1\) is zero.

  2. (ii)

    If \(f:V \rightarrow {\mathbb {N}}\) is such that \(f(m)\in \phi ^{-1}(m)\), then the asymptotic density of f(V) is 0. In particular, the asymptotic density of \(N_2\) and \(N_3\) is zero.

More generally, we also look at the Banach density of sets that are images of injective-increasing functions on \({\mathbb {N}}\).

Theorem 4

Let \(A \subset {\mathbb {N}}\). Suppose \(f:A\rightarrow {\mathbb {N}}\) is an injective and increasing function.

  1. (a)

    If the function \(\frac{f(n)}{n}\) is increasing on A and \(\lim \limits _{\begin{array}{c} n\rightarrow \infty \\ n \in A \end{array}}\frac{f(n)}{n} = \infty \), then the Banach density of f(A) is zero.

  2. (b)

    For \(A={\mathbb {N}}\), if there exists \(n_0\in {\mathbb {N}}\) and positive absolute constants \(c_1\) and \(c_2\) such that \(c_1n\le f(n)\le c_2n\) for \(n\ge n_0\), then the Banach density of \(f({\mathbb {N}})\) is positive.

In Theorem 4(a), the hypothesis ‘increasing’ for \(\frac{f(n)}{n}\) is only a sufficient condition. For instance, if \( BN_1 = \{m \in V : N_1(m) = N_2(m)\},\) then the function \(h :BN_1 \rightarrow N_1\) given by \(h(m)= N_1(m)\) does not satisfy this condition, but nevertheless, the Banach density of \(N_1\) is zero .

In section 3, we observe that \(N_2\supset N_1\) and \(N_3\supset {\mathbb {P}}\setminus \{2\}\), where \({\mathbb {P}}\) denotes the set of primes. Therefore, we look for infinite families of elements in \(N_2\setminus N_1\) and an infinite family of composite numbers in \(N_3\). This leads to our next theorem.

For \(r, r_1, r_2 \in {\mathbb {N}}\) and a prime \(q \equiv {3}\pmod {4}\), define

$$\begin{aligned} R(r_1, r_2)&:= 2.3^{r_1}.5^{r_2}, \ K_{q, r} := 2q^{r+1}, \\ k_{q,r}&:= {\left\{ \begin{array}{ll} q^{r}(q-1)+1 &{}\text{ if } q^{r}(q-1)+1 \in {\mathbb {P}}\\ q^{r+1} &{}\text{ otherwise. } \end{array}\right. } \end{aligned}$$

A prime of the form \(2^{2^l}+1\) with \(l\in {\mathbb {N}}\cup \{0\}\) is called a Fermat prime. We denote the j-th Fermat prime by \(F_j\). The only known Fermat primes are

$$\begin{aligned} F_1=3, \ F_2=5, \ F_3=17, \ F_4=257 \text { and } F_5=65537. \end{aligned}$$

The existence of \(F_6\) is not known.

Theorem 5

\({\mathcal {K}}_{\max }, {\mathcal {R}}\) and \({\mathcal {F}}\) are infinite subsets of \(N_2\) in which only finitely many elements are in \(N_1\). Moreover, \({\mathcal {K}}_{\min }\) is an infinite subset of \(N_3\) in which infinitely many elements are composite. Here,

where \(F_j\) denotes the \(j\)-th Fermat prime and \(H=\left\{ k \in {\mathbb {N}}:F_k \text { exists}\right\} \).

From this theorem, we observe that \(N_2\) contains infinitely many elements divisible by powers of n, where n is 2, 5 or a prime q with \(q \equiv 3\pmod {4}\). The infinite family \({\mathcal {F}}\) of elements of \(N_2\) shows the importance of Fermat primes to generate many elements in \(N_2\). The family \({\mathcal {K}}_{\min }\) of elements of \(N_3\) shows that \(N_3\) contains infinitely many elements divisible by powers of some prime q, where \(q\equiv {3}{\pmod {4}}\).

Though we have given examples of infinite families in \(N_2\) and \(N_3\), there may still be other elements in these sets. So, we give bounds for general \(N_2(m)\) and \(N_3(m)\) in the case when \(m \not \equiv {0}{\pmod {8}}\). We also study properties of the ratio \(\frac{N_2(m)}{N_3(m)}\) and geometric progressions contained inside \(N_2\) and \(N_3\). The ratios \(\frac{N_2(m)}{N_3(m)}\) are important in the sense that the statement ‘\(\frac{N_2(m)}{N_3(m)}>1\) for each \(m\in {\mathbb {V}}\)’ is equivalent to Carmichael’s conjecture which asserts that \(|\phi ^{-1}(m)|\ge 2\) for all \(m\in V\).

Theorem 6

Let \(m \in V\).

  1. (i)

    If \(m\equiv {2}{\pmod {4}}\) or \({4}{\pmod {8}}\), then \(m<N_3(m)<2m\) and \(2m<N_2(m)<4m.\)

  2. (ii)

    There exist infinitely many m such that \(\frac{N_2(m)}{N_3(m)}=2\). Further, if \(m\equiv 2 \pmod {4}\), then \(2 \le \frac{N_2(m)}{N_3(m)}\le 3\).

  3. (iii)

    \(N_2\) and \(N_3\) contain an infinite geometric progression.

In section 4, we discuss about the existence of arithmetic progressions in infinite subsets of natural numbers. The famous Szemerédi’s theorem [7] gives a sufficient condition for the existence of arbitrarily long arithmetic progressions in a subset of integers, namely, a positive asymptotic density. But this is not a necessary condition. Therefore we give a class of subsets of the integers having zero asymptotic density and containing arbitrarily long arithmetic progressions. These sets are formed by taking exactly one element from each pre-image \(\phi ^{-1}(m)\), \(m \in V\). Theorem 7 below follows as a consequence by using results due to Green and Tao [6] and Erdős [3, Theorem 4]. Therefore, we have as follows.

Theorem 7

If \(f:V \rightarrow {\mathbb {N}}\) is such that \(f(m)\in \phi ^{-1}(m)\), then f(V) contains arbitrarily long arithmetic progressions.

Indeed, we observe that these sets satisfy the hypothesis of the so-called Erdős–Turán conjecture [7, page 4] which asserts that if a set X of positive integers such that the sum of reciprocals of elements of X diverges, then X contains arbitrarily long arithmetic progressions.

Finally, in section 5, we pose some questions about elements of \(N_2\) and Banach density of \(N_2\) and \(N_3\) arising from the present work.

We use the following notation in this paper. Let \({\mathbb {N}}, {\mathbb {P}},\mathbb {R}^+\) and \({\mathbb {Z}}\) denote, respectively, the set of positive integers, the set of prime numbers, the set of positive real numbers and the set of integers. pq will always represent prime numbers unless otherwise mentioned. We write \(f(x)=o(g(x))\) if \(\frac{f(x)}{g(x)} \rightarrow 0\) as \(x \rightarrow \infty \). [ab] denotes the set \(\{x \in {\mathbb {N}}: a \le x \le b\}\) and similarly for the sets (ab], [ab) and (ab), and finally W(x) denotes the set of prime divisors of x. By convention, we assume empty products and empty sums to take the values 1 and 0 respectively. By ‘a divergent sequence \((x_n)\)’, we mean that \(x_n\rightarrow \infty \) as \(n\rightarrow \infty \).

2 Sparse subsets of natural numbers and sparsely totient numbers

It is well-known that the set of totients V is sparsely distributed, i.e., has asymptotic density zero (see, for example, [4] and references therein).

PROPOSITION 8

[4]

If V(x) is the number of totients less than or equal to x, then

$$\begin{aligned} V(x)=\displaystyle \frac{x}{\log x}\exp \{(C+o(1))(\log \log \log x)^2\}, \end{aligned}$$

where \(0.81<C<0.82.\)

Here, we study the sparseness of the set of totients V, the set of sparsely totient numbers \(N_1\) and other subsets of natural numbers using a generalized version of asymptotic density called the Banach density. We will define Banach density using Følner sequences.

DEFINITION 9

(Følner sequence)

A Følner sequence in a countable commutative semigroup \((G,+)\) is a sequence \((F_{n})_{n\in {\mathbb {N}}}\) of finite subsets of G such that for all \(g\in G\),

$$\begin{aligned} \displaystyle \lim _{n\rightarrow \infty }\displaystyle \frac{|(g+F_{n})\cap F_n|}{|F_n|}=1. \end{aligned}$$

Example 10

In the semigroup \(({\mathbb {N}},+)\), let \(F_n=[\alpha _n, \beta _n]\) with \(\beta _n - \alpha _n \rightarrow \infty \) as \(n \rightarrow \infty \), then \((F_{n})_{n\in {\mathbb {N}}}\) is a Følner sequence.

DEFINITION 11

(Density of a subset of \({\mathbb {N}}\))

Let \((F_{n})_{n\in {\mathbb {N}}}\) be a Følner sequence in \({\mathbb {N}}\) and \(A\subset {\mathbb {N}}\). Then the upper density of A with respect to the Følner sequence \((F_n)_{n\in {\mathbb {N}}}\) is defined by

$$\begin{aligned} {\overline{d}}_{F_n}(A)=\limsup _{n\rightarrow \infty }\displaystyle \frac{|F_n\cap A|}{|F_n|} \end{aligned}$$

and the lower density of A with respect to the Følner sequence \((F_n)_{n\in {\mathbb {N}}}\) is defined by

$$\begin{aligned} \underline{d}_{F_n}(A)=\liminf _{n\rightarrow \infty }\displaystyle \frac{|F_n\cap A|}{|F_n|} . \end{aligned}$$

If the upper density and the lower density are equal, then we say that the density of A with respect to the Følner sequence exists and it equals

$$\begin{aligned} d_{F_n}(A)=\lim _{n\rightarrow \infty }\displaystyle \frac{|F_n\cap A|}{|F_n|}. \end{aligned}$$

DEFINITION 12

(Asymptotic density)

The density with respect to the Følner sequence \(([1,n])_{n\in {\mathbb {N}}}\) is called asymptotic density. In this case, the upper asymptotic density, the lower asymptotic density and the density of a subset A are denoted by \({\overline{d}}(A)\), \(\underline{d}(A)\) and d(A) respectively.

DEFINITION 13

(Banach density)

The Banach density \(\displaystyle d^{*}(A)\) of \(A\subset {\mathbb {N}}\) is defined by

Example 14

Banach density of the set of primes is zero (see [5, p. 194]).

Using \(F_n=[1,n]\) for all \(n\in {\mathbb {N}}\) in the following proposition, one can observe that the Banach density of a subset of \({\mathbb {N}}\) is equal to density of that subset with respect to the Følner sequence \(([t_n+1, t_n+n])_{n\in {\mathbb {N}}}\) for some sequence \((t_n)_{n\in {\mathbb {N}}}\) in \({\mathbb {N}}\). Therefore, it is enough to consider Følner sequences formed by intervals in \({\mathbb {N}}\) to evaluate Banach density.

PROPOSITION 15

(p. 418, [1])

Given a subset A of \({\mathbb {N}}\) and any Følner sequence \((F_{n})_{n\in {\mathbb {N}}}\), there is a sequence \((t_n)_{n\in {\mathbb {N}}}\) such that

$$\begin{aligned} d^{*}(A)=d_{(F_n+t_n)}(A). \end{aligned}$$

2.1 Proof of Theorem 3

We now show that the Banach density of the set of totients V and that of the set of sparsely totient numbers \(N_1\) is zero. For this, we start with some necessary lemmas.

Lemma 16

If \(0<\alpha <\infty \) and \(x\ge 3\), then \(\log (1+\alpha )+\log x\le (1+\alpha )\log x\).

Proof

It is enough to show that \(f_{\alpha }(x) = x^{\alpha }-\alpha - 1 \ge 0\) for \(x \ge 3, \ 0< \alpha < \infty \). Since \(f_\alpha (x)\) is increasing in x, it suffices to show that \(f_{\alpha }(3) \ge 0\). Note that \(g(\alpha )= f_{\alpha }(3)\) is a strictly increasing function in \(\alpha \) and moreover, \(g(0)=0\). So \(g(\alpha ) > 0\) for \(\alpha > 0\). \(\square \)

Lemma 17

If \(0<\alpha \le 1\) and \(z\in {\mathbb {R}}^{+}\), then

$$\begin{aligned} \displaystyle \frac{\exp ((1+\alpha )^2z)-\exp (z)}{\alpha }\le \exp (4z)-1. \end{aligned}$$

Proof

By the definition of the exponential function, we have

$$\begin{aligned} \frac{\exp ((1+\alpha )^2z)-\exp (z)}{\alpha }&=\lim _{k\rightarrow \infty }\sum _{n=0}^{k}\left( \frac{(1+\alpha )^{2n}-1}{\alpha }\right) \frac{z^n}{n!}. \end{aligned}$$

Applying the binomial theorem, we get

$$\begin{aligned} \frac{\exp ((1+\alpha )^2z)-\exp (z)}{\alpha }&=\lim _{k\rightarrow \infty }\sum _{n=1}^{k}\left( \sum _{m=1}^{2n}{2n \atopwithdelims ()m} \alpha ^{m-1}\right) \frac{z^n}{n!} \\&\le \lim _{k\rightarrow \infty }\sum _{n=1}^{k}\left( \sum _{m=1}^{2n}{2n \atopwithdelims ()m} \right) \frac{z^n}{n!}, \end{aligned}$$

as \(\alpha \le 1.\) Again, referring to the binomial theorem, one has

$$\begin{aligned} \frac{\exp ((1+\alpha )^2z)-\exp (z)}{\alpha }\le \lim _{k\rightarrow \infty }\sum _{n=1}^{k}\frac{(4z)^n}{n!}=\exp (4z)-1. \end{aligned}$$

\(\square \)

Lemma 18

Suppose that \((F_n)_{n\in {\mathbb {N}}}\) is a Følner sequence on \({\mathbb {N}}\) defined by

$$\begin{aligned} F_n=(x_n,x_n(1+\alpha _n)], \end{aligned}$$

where \((x_{n})_{n=1}^\infty \) is a sequence in \({\mathbb {N}}\) and \((\alpha _{n})_{n=1}^\infty \) is a sequence of positive reals such that \(x_n\rightarrow \infty \) and \(\alpha _n\rightarrow 0\) as \(n\rightarrow \infty \). Then there exist \(n_0\in {\mathbb {N}}\) and \(0<k<1\) such that

$$\begin{aligned} \displaystyle \frac{|V\cap F_n|}{|F_n|}\le \displaystyle \frac{2}{(\log x_n)^{1-k}} \quad \forall \ n\ge n_0, \end{aligned}$$

so that \({\overline{d}}_{F_n}(V)=0\).

Proof

Since \(x_n\rightarrow \infty \) and \(\alpha _n\rightarrow 0\) as \(n\rightarrow \infty \), we can choose \(n_0\in {\mathbb {N}}\) such that \(\log \log \log x_n > 5\) and \(0<\alpha _n<1\) for all \(n\ge n_0\). Applying Lemma 16 for each \(n\ge n_0,\) we get

$$\begin{aligned} \log \log \log ((1+\alpha _n)x_n)&\le \log \log ((1+\alpha _n)\log x_n) \nonumber \\&\le \log ((1+\alpha _n)\log \log x_n) \nonumber \\&\le (1+\alpha _n)\log \log \log x_n . \end{aligned}$$
(1)

Using the estimate of V(x) from Proposition 8 (with the same constant C appearing there), equation (1) and setting \(z_n=(C+o(1))(\log \log \log x_n)^2\) for each \(n\ge n_0\), we get that for \(n\ge n_0\),

$$\begin{aligned} \displaystyle \frac{|V\cap F_n|}{|F_n|}&= \displaystyle \frac{|V\cap [1,x_n(1+\alpha _n)]|-|V\cap [1,x_n]|}{|F_n|} \\&\le \displaystyle \frac{1}{\alpha _n \log x_n}((1+\alpha _n)\exp ((1+\alpha _n)^2z_n)-\exp (z_n)). \end{aligned}$$

Since \(\alpha _n < 1\) and \(z_n \in \mathbb {R}^+\), then Lemma 17 gives

$$\begin{aligned} \displaystyle \frac{|V\cap F_n|}{|F_n|}&\le \displaystyle \frac{1}{\log x_n}\left( 2\exp (4z_n)-1\right) \\&\le \displaystyle \frac{2}{\log x_n}(\exp (4(C+o(1))(\log \log \log x_n)^2)). \end{aligned}$$

Applying \(4u^2< e^u\) for all \(u > 5\) with \(u= \log \log \log x_n\), we get

$$\begin{aligned} \displaystyle \frac{|V\cap F_n|}{|F_n|} \le \displaystyle \frac{2}{\log x_n}(\exp ((C+o(1))\log \log x_n)) \le \displaystyle \frac{2(\log x_n)^{C+o(1)}}{\log x_n}. \end{aligned}$$

Since \(C < 1\), choose a sufficiently large \(n_1\) with \(n_1\ge n_0\) such that \(|C+o(1)|<k\) for all \(n\ge n_1\) and for some \(0<k<1\). Therefore,

$$\begin{aligned} \displaystyle \frac{|V\cap F_n|}{|F_n|}\le \displaystyle \frac{2}{(\log x_n)^{1-k}}\quad \forall ~ n\ge n_1 \end{aligned}$$

and hence \({{\bar{d}}}_{F_n}(V)=0\). \(\square \)

Lemma 19

Suppose that \((F_n)_{n\in {\mathbb {N}}}\) is a Følner sequence on \({\mathbb {N}}\) defined by

$$\begin{aligned} F_n=(x_n,x_n(1+\alpha _n)], \end{aligned}$$

where \((x_{n})_{n=1}^\infty \) is a sequence in \({\mathbb {N}}\) and \((\alpha _{n})_{n=1}^\infty \) is a sequence of positive reals such that \(\alpha _n>\alpha _0>0\) for each \(n \in {\mathbb {N}}\). Then \({{\bar{d}}}_{F_n}(V)=0\).

Proof

Since \(|F_n|=x_n\alpha _n\rightarrow \infty \) as \(n\rightarrow \infty \), we can choose \(n_0\in {\mathbb {N}}\) such that \(\log \log \log (1 + \alpha _n)x_n>0\) for all \(n\ge n_0\). For \(n \ge n_0\), we get

$$\begin{aligned} \displaystyle \frac{|V\cap F_n|}{|F_n|}&=\displaystyle \frac{|V\cap [1,x_n(1+\alpha _n)]|-|V\cap [1,x_n]|}{|F_n|} \le \displaystyle \frac{|V\cap [1,x_n(1+\alpha _n)]|}{|F_n|}. \end{aligned}$$

Using the estimate of V(x) from Proposition 8 (with the same constant C appearing there), we get

$$\begin{aligned} \frac{|V\cap F_n|}{|F_n|}&\le \displaystyle \frac{(1+\alpha _n)}{\alpha _n \log ((1+\alpha _n) x_n)}(\exp ((C+o(1))(\log \log \log (1+\alpha _n)x_n)^2)). \end{aligned}$$

Since \(\alpha _n > \alpha _0\) for each \(n \in {\mathbb {N}}\), applying the inequality \(y^2 < e^y\) for \(y >0\) gives us

$$\begin{aligned} \frac{|V\cap F_n|}{|F_n|}&\le \displaystyle \frac{(1+\alpha _0)}{\alpha _0}\left( \displaystyle \frac{\left( \log ((1+\alpha _n) x_n)\right) ^{C+o(1)}}{\log ((1+\alpha _n) x_n)}\right) \rightarrow 0 \text { as } n\rightarrow \infty , \end{aligned}$$

as \(C<1\). Hence \({\overline{d}}_{F_n}(V)=0\). \(\square \)

PROPOSITION 20

The Banach density of the set of totients is zero.

Proof

Let \((F_n)_{n\in {\mathbb {N}}}\) be a Følner sequence on \({\mathbb {N}}\) defined by \(F_n=(x_n,x_n(1+\alpha _n)]\), where \(x_n \in {\mathbb {N}},\ \alpha _n \in {\mathbb {R}}^{+}\). To prove that the Banach density of V is zero, it is enough to show that \({{\bar{d}}}_{F_n}(V)=0\) in the following cases:

Case A. :

\(\alpha _n \rightarrow 0\) and \(x_n\rightarrow \infty \) as \(n\rightarrow \infty .\)

Case B. :

There exists \(\alpha _0>0\) such that \(\alpha _n>\alpha _0\) for each \(n\in {\mathbb {N}}.\)

For Case A, the result follows from Lemma 18 and similarly, Lemma 19 covers Case B.   \(\square \)

Next, we proceed to prove that the Banach density of \(N_1\) is zero.

Lemma 21

Suppose \(A,B\subset {\mathbb {N}}\) and \(g:A\rightarrow B\) is an injective map satisfying \(g(x)\le x\) for all \(x\in A\). Let \((F_n)_{n\in {\mathbb {N}}}\) be a Følner sequence in \({\mathbb {N}}\) such that \(F_n=(a_n, x_n]\) and \((a_{n})_{n=1}^\infty \) is bounded. Then \({{\bar{d}}}_{F_n}(B)=0\) implies \({{\bar{d}}}_{F_n}(A)=0\).

Proof

Since \(g:A\rightarrow B\) is an injective map and \(g(x)\le x\) for all \(x\in A\), we have \(g: F_n\cap A\rightarrow g(A)\cap [1,x_n]\) is injective for each \(n\in {\mathbb {N}}\). It follows that \(|F_n\cap A|\le |g(A)\cap [1,x_n]|\) for all \(n\in {\mathbb {N}}\). Since \( g(A)\cap [1,x_n]\subset \left( g(A)\cap F_n\right) \cup [1,a_n]\), we get that \(|F_n\cap A|\le |g(A)\cap F_n|+ |[1,a_n]|\). Therefore,

$$\begin{aligned} \frac{|F_n\cap A|}{|F_n|}\le \frac{|F_n\cap g(A)|}{|F_n|}+\frac{a_n}{|F_n|}\le \frac{|F_n\cap g(A)|}{|F_n|}+\frac{a}{|F_n|}, \end{aligned}$$

where a is an upper bound of the sequence \((a_{n})_{n=1}^\infty \). Since \(g(A) \subset B\) and \({{\bar{d}}}_{F_n}(B)=0\), we conclude that \({{\bar{d}}}_{F_n}(A)=0.\) \(\square \)

COROLLARY 22

Let \((F_n)_{n\in {\mathbb {N}}}\) be a Følner sequence on \({\mathbb {N}}\) defined by \(F_n=(a_n,x_n],\) where \((a_{n})_{n=1}^\infty \) is bounded. If \(f:V \rightarrow {\mathbb {N}}\) is such that \(f(m)\in \phi ^{-1}(m),\) then \({{\bar{d}}}_{F_n}(f(V))=0\). In particular,  the asymptotic density of \(N_1,N_2\) and \(N_3\) is zero.

Proof

Consider \(g :f(V)\rightarrow V\) defined by \(g(n) = \phi (n)\). This is an injective map satisfying \(g(x)\le x\) \(\forall x \in f(V)\). Since \(d^*(V)=0\), by Proposition 20, it follows that \({{\bar{d}}}_{F_n}(f(V))=0\), by applying Lemma 21. In particular, \({{\bar{d}}}_{F_n}(N_i)=0\) for \(i= 2, 3\). Also, \(N_1\subset N_2\) so that \({{\bar{d}}}_{F_n}(N_1)=0\). \(\square \)

PROPOSITION 23

The Banach density of \(N_1\) is zero.

Proof

Let \((F_n)_{n\in {\mathbb {N}}}\) be a Følner sequence on \({\mathbb {N}}\) defined by \(F_n=(x_n,x_n+y_n],\) where \((x_{n})_{n=1}^\infty \,\text {and}\,(y_{n})_{n=1}^\infty \) are sequences in \({\mathbb {N}}\) with \(y_n\rightarrow \infty \). To show that the Banach density of \(N_1\) is zero, it is enough to prove that \({{\bar{d}}}_{F_n}(N_1)=0\) for the two cases when (i) \((x_{n})_{n=1}^\infty \) is bounded, and (ii) \((x_{n})_{n=1}^\infty \) is divergent.

If \((x_{n})_{n=1}^\infty \) is a bounded sequence, then Corollary 22 establishes that \({{\bar{d}}}_{F_n}(N_1)=0\). So we can assume that \((x_{n})_{n=1}^\infty \) is a divergent sequence. If p is a prime number, then Proposition 2(ii) gives the existence of an element \(m_0(p)\in {\mathbb {N}}\) such that \(N_1(m)\equiv 0 \pmod {p}\) for each \(m\ge m_0(p)\). Since \((x_{n})_{n=1}^\infty \) is divergent, we can choose \(n_0(p)\in {\mathbb {N}}\) such that \(x_n>N_1(m_0(p))\) for each \(n>n_0(p)\). Then \(F_n\) contains at most \( \frac{|F_n|}{p}+1\) elements of \(N_1\) for \(n>n_0(p)\). So, given a prime p, there exists \(n_0(p)\in {\mathbb {N}}\) such that

$$\begin{aligned} n>n_0(p)\Rightarrow \displaystyle \frac{|F_n\cap N_1|}{|F_n|}\le \displaystyle \frac{1}{p} + \frac{1}{y_n} . \end{aligned}$$

Since \(y_n \rightarrow \infty \) as \(n \rightarrow \infty \), this means that \({{\bar{d}}}_{F_n}(N_1) \le \frac{1}{p}\). As this holds for each prime p, we conclude that \({{\bar{d}}}_{F_n}(N_1)=0\) if \((x_{n})_{n=1}^\infty \) is divergent. Therefore, \(d^*(N_1)=0\). \(\square \)

Hence, the proof of Theorem  3 is complete by collecting Proposition 23, Proposition 20 and Corollary 22.

2.2 Some criteria for sparse sets in \({\mathbb {N}}\)

We have studied the Banach densities of specific sets like V and \(N_1\). Now, we are going to investigate the behavior of sparse sets which are the images of injective, increasing functions on \({\mathbb {N}}\). We proceed to the proof of Theorem 4.

Proof of Theorem 4(a)

Let (ab] be an interval in \({\mathbb {N}}\) such that \(a,b\in f(A)\). Since f is injective and increasing, it follows that \(\{x\in A:f(x)\in (a,b]\}= \{x\in A:x\in (f^{-1}(a),f^{-1}(b)]\}\). Therefore,

$$\begin{aligned} \displaystyle \frac{|(a,b]\cap f(A)|}{b-a} \le \displaystyle \frac{f^{-1}(b)-f^{-1}(a)}{b-a}= \displaystyle \frac{1}{b-a}\left( \frac{b}{m_b}-\frac{a}{m_a}\right) , \end{aligned}$$

where \(m_a=\displaystyle \frac{a}{f^{-1}(a)}\) and \(m_b=\displaystyle \frac{b}{f^{-1}(b)}\). Since the function \(\displaystyle \frac{f(n)}{n}\) is increasing on A, we get \(m_a\le m_b\). It follows that

$$\begin{aligned} \displaystyle \frac{|(a,b]\cap f(A)|}{b-a}\le \displaystyle \frac{1}{m_b}. \end{aligned}$$

Suppose that \(((a_n,b_n])_{n\in {\mathbb {N}}}\) is a Følner sequence with \(|(a_n, b_n]\cap A |>1\). Then for each \(n\in {\mathbb {N}}\), there exist \(a_n',b_n' \in f(A)\) such that \((a_n,b_n]\cap f(A)=[a_n',b_n']\cap f(A)\) and \(a_n<a_n'< b_n'\le b_n\). So,

$$\begin{aligned} \limsup _{b_n-a_n\rightarrow \infty }\displaystyle \frac{|(a_n,b_n]\cap f(A)|}{b_n-a_n}\le \limsup _{b_n-a_n\rightarrow \infty }\displaystyle \frac{|(a_n',b_n']\cap f(A)|}{b_n'-a_n'}\le \limsup _{b_n-a_n\rightarrow \infty } \displaystyle \frac{1}{m_{b_n'}}. \end{aligned}$$

Note that \(b_n-a_n\rightarrow \infty \Rightarrow b_n \rightarrow \infty .\) We claim that \(b_n ' \rightarrow \infty \). If not, there exists a subsequence \(\{b_{n_k}'\}\) of \(\{b_n'\}\) such that \(b_{n_k}' \le l\) for all \(k \in {\mathbb {N}}\). By the definition of \(b_n'\), we know that \((b_{n_k}', b_{n_k}] \cap f(A) = \varnothing \) for each k. In particular, \((l, b_{n_k}] \cap f(A)= \varnothing \) for all k. Since \(b_{n_k} \rightarrow \infty \) as \(k \rightarrow \infty \), this implies that \(f(x) \le l\) for all \(x \in A\). But this is a contradiction since f is strictly increasing and hence grows indefinitely. Thus, \(b_n ' \rightarrow \infty \). Therefore,

$$\begin{aligned} \limsup _{b_n-a_n\rightarrow \infty }\displaystyle \frac{|(a_n,b_n]\cap f(A)|}{b_n-a_n}\le \limsup _{b_n' \rightarrow \infty } \displaystyle \frac{1}{m_{b_n^{'}}}=0, \end{aligned}$$

since \(({m_{b_n'}})_{n=1}^{\infty }\) is a subsequence of the divergent sequence \(\left( \frac{f(n)}{n}\right) _{n\in A}\).

Hence, \({{\bar{d}}}_{F_n}(f(A))=0\) for all Følner sequences \((F_{n})_{n\in \mathbb {N}}\) with \(F_n=(a_n, b_n]\). So, \(d^{*}(f(A))=0.\) \(\square \)

Proof of Theorem 4(b)

Choose \(r\in {\mathbb {N}}\setminus \{1\}\) such that \(c_2<(r-1)c_1\) and consider the Følner sequence \((F_n)_{n\in {\mathbb {N}}}\) given by \(F_n=(n,rn]\). Since \(f(\mathbb {N})\) is an infinite set, there exists \(k^\prime \in \mathbb {N}\) that for each \(n \ge k^{\prime }\), one can choose integers \(a_n, b_n \in f({\mathbb {N}})\) such that \((a_n,b_n)\cap f({\mathbb {N}})=(n,rn] \cap f({\mathbb {N}})\) and \(a_n\le n< rn\le b_n\). Since f is injective and increasing, it follows that \(\{x\in {\mathbb {N}}:f(x)\in (a_n,b_n)\}= \{x\in {\mathbb {N}}:x\in (f^{-1}(a_n),f^{-1}(b_n))\}\). We now have

$$\begin{aligned} \displaystyle \frac{|F_n\cap f({\mathbb {N}}) |}{|F_n|}&= \displaystyle \frac{f^{-1}(b_n)-f^{-1}(a_n)-1}{(r-1)n}. \end{aligned}$$
(2)

From the hypothesis, we have

$$\begin{aligned} c_1x\le f(x)\le c_2 x \quad \forall \ x\ge n_0. \end{aligned}$$
(3)

Choose an integer \(k \ge k^\prime \) such that \(f^{-1}(a_n),f^{-1}(b_n)\ge n_0\) for all \(n\ge k\). Inserting the values of \(f^{-1}(a_n), f^{-1}(b_n)\) obtained from inequality (3) in equation (2) for \(n\ge k\), we get

$$\begin{aligned} \displaystyle \frac{|F_n\cap f({\mathbb {N}}) |}{|F_n|}&\ge \displaystyle \frac{1}{(r-1)n}\left( \displaystyle \frac{b_n}{c_2}-\displaystyle \frac{a_n}{c_1}\right) -\frac{1}{(r-1)n}\\&> \displaystyle \frac{1}{(r-1)n}\left( \displaystyle \frac{rn}{(r-1)c_1}-\displaystyle \frac{n}{c_1}\right) \\&\quad -\frac{1}{(r-1)n} \ \ (\text {since } c_2<(r-1)c_1)\\&=\displaystyle \frac{1}{c_1(r-1)^2}-\frac{1}{(r-1)n}. \end{aligned}$$

Therefore,

$$\begin{aligned} \limsup _{n \rightarrow \infty } \frac{|F_n\cap f({\mathbb {N}}) |}{|F_n|} \ge \frac{1}{c_1(r-1)^2}. \end{aligned}$$

Hence, \({{\bar{d}}}_{F_n}(f({\mathbb {N}}))>0\) and therefore \(d^{*}(f({\mathbb {N}}))>0\). \(\square \)

We drop the ‘increasing function’ hypothesis on \(\frac{f(n)}{n}\) in Theorem 4(a) and by the two examples given below, we show that the conclusion on Banach density may or may not hold.

Example 24

Given \(k,l\ge 2\), we define a map \(f_{k,l}:{\mathbb {N}}\rightarrow {\mathbb {N}}\) by

$$\begin{aligned} f_{k,l}(x)= {\left\{ \begin{array}{ll} k^{2nl}+(l-1)x &{}\text{ if } k^{2n}\le x< k^{2n+1},\\ x^{l} &{}\text{ if } k^{2n+1}\le x<k^{2n+2}. \end{array}\right. } \end{aligned}$$

One can see that \(f_{k, l}\) is injective and increasing. The function \(\frac{f_{k,l}(n)}{n}\) is divergent but not increasing. Note that \(f_{k, l}([k^{2n}, k^{2n + 1}))\) is an arithmetic progression of length \(k^{2n + 1} - k^{2n}\) with a common difference \(l-1\). Therefore, \(f_{k,l}({\mathbb {N}})\) contains arbitrarily long arithmetic progressions with a common difference \(l-1\). Hence, it has positive Banach density.

The above example suggests that the ‘increasing’ hypothesis on \(\frac{f(n)}{n}\) is necessary. However, this is not always the case as the next example shows.

Suppose \(BN_1=\{m \in V :~N_1(m)=N_2(m)\}\). Then the function \(h:BN_1\rightarrow N_1\) defined by \(h(m)=N_1(m)\), is both bijective and increasing. By a result of Sanna [11, Lemma 2.1] on the asymptotic of \(N_1(m)\), it is readily seen that \(\frac{h(m)}{m}=\frac{N_1(m)}{m} \rightarrow \infty \) as \(m \rightarrow \infty .\)

Lemma 25

[11, Lemma 2.1]. \(N_1(m)\sim e^{\gamma }\) \(m\log \log m\) as \(m\rightarrow \infty ,\) where \(\gamma \) is the Euler–Mascheroni constant.

The next proposition tells us that \(\frac{h(m)}{m}\) is not an increasing function.

PROPOSITION 26

\(\displaystyle \frac{h(n)}{n}:BN_1\rightarrow N_1\) is not an increasing function.

Proof

For \(p\in {\mathbb {P}}\), define

$$\begin{aligned} X_{p}=\prod _{q\in {\mathbb {P}},q\le p}q. \end{aligned}$$

Let \(p_1\) and \(p_2\) be two consecutive primes such that \(3<p_1<p_2\). Let \(a=X_{p_1}\), \(b=\frac{X_{p_2}}{p_1}\), \(M_{a}=\phi (a)\) and \(M_{b}=\phi (b)\). Then by Proposition 1 and the definition of h, we get

$$\begin{aligned} h(M_a)=N_1(M_a)=a \quad \text { and }\quad h(M_b)=N_1(M_b)=b. \end{aligned}$$

The proof of the equation \(h(M_b)=b\) uses a result of Nagura which states that \((n, 1.2n) \cap \mathbb {P} \ne \emptyset \, \text {for all} \ n>25.\) This gives

$$\begin{aligned} \displaystyle \frac{h(M_a)}{M_a}=\displaystyle \prod _{q\in {\mathbb {P}}, q\le p_1}\frac{q}{q-1}=\left( \frac{p_1}{p_1-1}\right) \left( \frac{p_2-1}{p_2}\right) \frac{h(M_b)}{M_b}. \end{aligned}$$

Since \(p_1<p_2\), it follows that

$$\begin{aligned} \displaystyle \frac{h(M_a)}{M_a}>\frac{h(M_b)}{M_b} \ \text { but}\ M_{a}=\frac{p_1-1}{p_2-1}M_b<M_b. \end{aligned}$$

Therefore, \(\frac{h(n)}{n}\) is not an increasing function. \(\square \)

In contrast to \(f_{k, l}\), though \(\frac{h(n)}{n}\) is not increasing, we know that \(d^{*}(h(BN_1))=d^{*}(N_1)=0\). Therefore, we observe that if we remove the condition of ‘increasing map’ on \(\frac{f(n)}{n}\), then both the possibilities, namely Banach density is zero or positive may occur.

3 Explicit construction of elements of \(N_2\) and \(N_3\)

3.1 Proof of Theorem 5

Now we move on to the study of \(N_2\) and \(N_3\). As we know that \(N_1 \varsubsetneq N_2\), we give explicit examples of infinite families of elements in \(N_2 \setminus N_1\). Since \(\phi (p)=p-1\) and \(\phi (p-1)<p-1\) for an odd prime p, this implies that \(N_3(p-1)=p\). So, \({\mathbb {P}}\setminus \{2\} \subset N_3.\) We are going to show that infinitely many composite numbers also lie in \(N_3\). First, we give the following useful definitions.

DEFINITION 27

(\(k_{q,r}\) and \(K_{q,r}\))

For \(q \in {\mathbb {P}}\) and \(r, r_1, r_2 \in {\mathbb {N}}\), define

$$\begin{aligned} k_{q,r}&:= {\left\{ \begin{array}{ll} q^{r}(q-1)+1 &{}\text{ if } q^{r}(q-1)+1 \text { is a prime},\\ q^{r+1} &{}\text{ otherwise } . \end{array}\right. } \\ K_{q,r}&:= 2q^{r+1}, R(r_1,r_2) = 2\cdot 3^{r_1}\cdot 5^{r_2}. \end{aligned}$$

The following lemma gives a description of the elements of \(\phi ^{-1}(m)\) for \(m \equiv 2 \pmod {4}.\) This will be useful to construct families of elements in \(N_2\) and \(N_3\) as indicated above.

Lemma 28

Let A(m) denote the number of solutions to the equation \(\phi (x)=m.\) Suppose \(m>2\) and \(m \equiv 2 \pmod {4}\). Then

  1. (i)

    every element of \(\phi ^{-1}(m)\) is of the form \(p^{\alpha }\) or \(2p^{\alpha }\), where \(p \equiv 3 \pmod {4};\)

  2. (ii)

    \(A(m) = 0, 2\) or 4; 

  3. (iii)

    if \(A(m) = 2,\) then \(\phi ^{-1}(m) = \{p^{\alpha }, 2p^{\alpha }\}\) for some \(p \equiv 3 \pmod {4},\) \(\alpha \ge 1\) and if \(A(m) = 4,\) then \(\phi ^{-1}(m) = \{p^{\beta }, 2p^{\beta }, q, 2q\}\) for some \(p, q \equiv 3 \pmod {4}, \text { with } p < q, \ \beta > 1.\)

Proof

For the proof of (i) and (ii), see [9]. For (iii), if \(p \in {\mathbb {P}},\ p \equiv 3 \pmod {4}\), then \(\phi (p^{\alpha })=\phi (2p^{\alpha })\) for \(\alpha \ge 0.\) If \(A(m)=2\), then from (i), \(\phi ^{-1}(m)= \{p^{\alpha }, 2p^{\alpha }\}\) for some prime \(p \equiv 3 \pmod {4}.\) On the other hand, if \(A(m)=4\), then \(\phi ^{-1}(m) = \{p^{\beta }, 2p^{\beta }, q^{\gamma }, 2q^{\gamma }\}\) for some \(p, q \equiv 3 \pmod {4}\) and \(\beta , \gamma \ge 1.\) Now, \(p^{\beta }\ne q^{\gamma }\) and \(\phi (p^{\beta })=\phi (q^{\gamma }) \Rightarrow p \ne q\). Without loss of generality, let us assume that \(p < q\). This means that \(\beta > \gamma .\) Now, \(\phi (p^{\beta })=\phi (q^{\gamma }) \Rightarrow p^{\beta - 1}(p-1)=q^{\gamma -1}(q-1)\). If \(\gamma > 1\), then it means that \(q \mid (p-1),\) a contradiction to \(p < q\). Thus, \(\gamma =1.\)

Therefore, \(\phi ^{-1}(m) = \{p^{\beta }, 2p^{\beta }, q, 2q\}\) for some \(p, q \equiv 3 \pmod {4} \text { with } p < q, \ \beta > 1\) in the case \(A(m)=4\). \(\square \)

Lemma 29

Let q be a prime greater than 7. Then there exists a unique odd integer \(n\in \{q+2, q+4\}\) such that \(n\equiv 0\pmod {3},\) \(\gcd (n,q)=1\) and \(\phi (n)< q\).

Proof

Since q is a prime and \(q>7\), one can choose the unique integer \(n\in \{q+2, q+4\}\) such that \(n\equiv 0\pmod {3}\). Since q is odd, \(\gcd (q,n)=1\). Let \(n=3^rl\) with \(r,l\in {\mathbb {N}}\) and \(3 \not \mid l\). Hence \(\phi (n)=2\times 3^{r-1}\phi (l)\le 2\times 3^{r-1}l=\frac{2n}{3}\le \frac{2q+8}{3}<q\) if \(q\ge 11\). \(\square \)

PROPOSITION 30

Suppose that \(r \in {\mathbb {N}}\) and q is a prime satisfying \(q\equiv 3 \pmod {4}\). Then

  1. (i)

    \(N_2(q^r(q-1))=K_{q,r}\) and \(N_3(q^r(q-1))= k_{q,r};\)

  2. (ii)

    \(K_{q,r} \notin N_1\) except when \((q,r)= (3,1)\).

Proof

Let \(m=q^{r}(q-1)\). Then \(q\equiv 3\pmod {4}\Rightarrow m\equiv 2 \pmod {4}\). Since \(\phi (q^{r+1})=q^{r}(q-1)\), it follows that \(\phi ^{-1}(m)\) is non-empty. Applying Lemma 28, we get \(\phi ^{-1}(m)=\{q_1^{\alpha }, 2q_1^{\alpha }\}\) or \(\{q_2^{\beta }, 2q_2^{\beta }, q_3, 2q_3\}\), where \(q_2<q_3\), \(m>2, \ \alpha \ge 1\) and \(\beta >1\).

If \(\phi ^{-1}(m)=\{q_1^{\alpha }, 2q_1^{\alpha }\}\), then \(q_1=q\) and \(\alpha =r+1\) as \(\phi (q^{r+1})=q^{r}(q-1)= \phi (2q^{r+1})\). Hence \(N_2(q^r(q-1))=K_{q,r}\) and \(N_3(q^r(q-1))= q^{r+1}\) in this case. Since \(\phi ^{-1}(m)\) does not contain primes, this means that \(m+1=q^r(q-1)+1\) is composite and hence \(k_{q, r}=q^{r+1}=N_3(m).\)

If suppose \(\phi ^{-1}(m)=\{q_2^{\beta }, 2q_2^{\beta }, q_3, 2q_3\}\), where \(q_2, q_3 \equiv 3 \pmod {4}, q_2<q_3\) and \(\beta >1\). If \(q^{r}(q-1)+1 \text { is a prime}\), then \(q^{r}(q-1)+1 \in \phi ^{-1}(m)\). It follows that \(N_3(q^r(q-1))= k_{q,r}\) in this case. Since the only prime in \(\phi ^{-1}(m)\) is \(q_3\), we get \(q_3=q^{r}(q-1)+1=N_3(q^r(q-1))\). This means that \(q_2^{\beta }>q_3\). Now, note that \(q^{r+1}> q^r(q-1) + 1\) and \(q^{r+1}\) is the only odd composite number in \(\phi ^{-1}(m)\). Thus, \(q_{2}^{\beta }=q^{r+1}\), i.e. \(q_2=q\) and \(\beta =r + 1\). Therefore, \(N_2(q^r(q-1))=2q^{r + 1}\). On the other hand, if \(q^{r}(q-1)+1 \text { is not a prime}\), then no element of \(\phi ^{-1}(m)\) can be prime. But this contradicts the fact that \(q_3\in \phi ^{-1}(m)\). Therefore,

$$\begin{aligned} N_2(q^r(q-1))=K_{q,r} \quad \text { and }\quad N_3(q^r(q-1))= k_{q,r}. \end{aligned}$$

Coming to the proof of (ii), if \(q> 7\), then Lemma 29 gives the odd integer \(n\in \{q+2, q+4\}\) such that \(n\equiv 0\pmod {3}\), \(\gcd (n,q)=1\) and \(\phi (n)< q\). Now we observe that \(2nq^{r-1}>2q^{r}\) but \(\phi (2nq^{r-1})\le \phi (2q^{r})\). Hence \(2q^r \not \in N_1\) for all \(q>7\). If \(q=7\), then \(2\times 3^2\times 7^{r-1}>2\times 7^r\) but \(\phi (2\times 3^2\times 7^{r-1})\le \phi (2\times 7^r)\) and hence \(2\times 7^r \not \in N_1\) for all \(r\ge 1\). If \(q=5\), then \(12\times 5^{r-1}>2\times 5^r\) but \(\phi (12\times 5^{r-1})\le \phi (2\times 5^r)\). Hence \(2\times 5^r \not \in N_1\) for all \(r\ge 1\). Since \(\phi (20\times 3^{r-2})<\phi (2\times 3^r)\) but \(20\times 3^{r-2}>2\times 3^r\) for \(r\ge 3\), it follows that \(2\times 3^r\not \in N_1\) for all \(r\ge 3. \) \(\square \)

From Proposition 30, we see that \(K_{q, r} \in N_2\setminus N_1\) for all \(r \ge 3\). So, for each \(q \equiv 3 \pmod {4}\), this gives an infinite family of elements in \(N_2 \setminus N_1\). But the proposition does not ensure the presence of infinitely many composite numbers in \(N_3\). For this, we require that \(k_{q, r}\) is composite for infinitely many (qr). If \(q=3\), note that \(2\cdot 3^r + 1\) is divisible by 5 when \(r \equiv 3 \pmod {4}\). In other words, \(k_{3, r}\) is composite for infinitely many r. So, from Proposition 30, we see that \(N_3\) contains infinitely many composite numbers.

Now, we give another infinite family of elements in \(N_2\setminus N_1\). First, we state some definitions, four preliminary lemmas and then prove the two main lemmas which together construct an infinite two-parameter family of elements in \(N_2\setminus N_1\).

DEFINITION 31

(D(AB))

Let A and B be two finite subsets of \({\mathbb {P}}\). Then D(AB) is defined by

$$\begin{aligned} D(A,B):=\left( \prod _{q\in A}\frac{q-1}{q}\right) \left( \prod _{q\in B}\frac{q}{q-1}\right) . \end{aligned}$$

Lemma 32

Suppose that \(y,x\in {\mathbb {N}}\setminus \{1\}\). If \(\phi (y)\le \phi (x)\) and \(y>x,\) then \(D(W(y),W(x))<1\).

Proof

We know that

$$\begin{aligned} \phi (y)=y\displaystyle \prod _{q\in W(y)}\left( \displaystyle \frac{q-1}{q}\right) \quad \text { and }\quad \phi (x)=x\displaystyle \prod _{q\in W(x)}\left( \displaystyle \frac{q-1}{q}\right) . \end{aligned}$$

This gives

$$\begin{aligned} 1\ge \displaystyle \frac{\phi (y)}{\phi (x)}=\displaystyle \frac{y D(W(y),W(x))}{x}>D(W(y),W(x)), \end{aligned}$$

since \(\phi (y)\le \phi (x)\) and \(y>x\). \(\square \)

Lemma 33

Let A and B be two finite subsets of \({\mathbb {P}}\) such that \(|B|\le |A|\). If \(\min (B\setminus A)>\max (A)\) or \(B\subset A,\) then \(D(B,A)\ge 1\).

Proof

If \(B \subset A\), then \(D(B, A) = \left( \prod _{q \in A\setminus B}\frac{q}{q-1}\right) \ge 1\). In the case when \(B \not \subset A\), define an injective map \(f:B\rightarrow A\) such that \(f(x)=x\) for \(x\in A\cap B\). If \(\min (B\setminus A)>\max (A)\), it follows that \(f(x)\le x\) for all \(x\in B\). Therefore,

$$\begin{aligned} D(B,A)\ge \prod _{x\in B}\left( \frac{(x-1)f(x)}{x(f(x)-1))}\right) =\prod _{x\in B}\left( \frac{xf(x)-f(x)}{xf(x)-x}\right) \ge 1. \end{aligned}$$

\(\square \)

Lemma 34

Let \(a,k\in {\mathbb {N}}\setminus \{1\}\) and \(k\le a\). Suppose that \( x_1, x_2, \dots , x_k\) are non-negative integers such that at least two of them are postive. Then

$$\begin{aligned} \sum _{i=1}^{k}a^{x_i}\le a^{x_1+x_2+\dots +x_k}. \end{aligned}$$

Proof

Since atleast two of the k integers \( x_1, x_2, \dots , x_k\) are positive, it follows that \(a^{x_i}\le a^{x_1+x_2+\dots +x_k-1}\) for all \(i \in [1,k]\). Therefore,

$$\begin{aligned} \sum _{i=1}^{k}a^{x_i}\le ka^{x_1+x_2+\dots +x_k-1}\le a^{x_1+x_2+\dots +x_k}, \end{aligned}$$

since \(k \le a\). \(\square \)

Lemma 35

Let \(x,y,k\in {\mathbb {N}}\) such that \(x,y,k \ge 2\) and \(k\le \min \{x,y\}\). Suppose that for each \(i\le k\), \(a_i\) and \(b_i\) are non-negative integers such that \(a_i+b_i\ne 0\). If \(a_1+a_2+\dots +a_k=t\) and \(b_1+b_2+\cdots +b_k=u,\) then

$$\begin{aligned} \sum _{i=1}^{k}x^{a_i}y^{b_i}\le x^ty^u. \end{aligned}$$

Proof

Let \(k \ge 2\). If possible, suppose that both the sequences \((a_i)_{i=1}^{k}\), \((b_i)_{i=1}^{k}\) contain at most one positive integer. If \(k \ge 3\), then there exists \(j \in [1, k]\) such that \(a_j + b_j = 0\), a contradiction. Hence, \(k=2\) and exactly one of \(\{a_1, a_2\}\) and one of \(\{b_1, b_2\}\) are positive with \(a_i + b_i \ne 0\) for \(i \in [1, 2]\). Therefore, we can assume \(a_1, b_2>0\), without loss of generality. We need to show that

$$\begin{aligned} x^{a_1} + y^{b_2} \le x^{a_1}y^{b_2} \end{aligned}$$

in this case. Since \(x, y \ge 2\) and \(a_1, b_2 \in {\mathbb {N}}\), it is enough to show that \(v+w \le vw\) for \(v,w \in {\mathbb {N}}\setminus \{1\}\). This happens iff \(v \le w(v-1)\) iff \(v/(v-1) \le w\) which is true since the left-hand side is not greater than 2.

On the other hand, if one of the sequences, say \((a_i)\), has at least two positive elements, then by Lemma 34, we have

$$\begin{aligned} \sum _{i=1}^{k}x^{a_i}y^{b_i} \le \left( \sum _{i=1}^{k}x^{a_i}\right) y^u \le x^{t}y^{u}, \end{aligned}$$

since \(k\le \min \{x,y\} \le x\). \(\square \)

DEFINITION 36

(Valuation)

Let p be a prime number. Then the p-valuation on the integers \({\mathbb {Z}}\) is the map \(v_{p} :{\mathbb {Z}}\rightarrow {\mathbb {N}}\cup \{0, \infty \}\) defined by \(v_{p}(0)=\infty \) and \(v_{p}(n)=r\) for \(n \ne 0\), where r is the largest non-negative integer such that \(p^{r} \mid n.\)

Lemma 37

Suppose \(r_1, r_2, y \in {\mathbb {N}}\) satisfy \(\phi (y)=\phi (R(r_1,r_2)),\ |W(y)|=4,\) \(v_2(y)=1,\) \(v_3(y)=0\) and \(v_5(y)=0\). Then \(y\le R(r_1,r_2)\).

Proof

Since \(|W(y)|=4\), \(v_2(y)=1\), \(v_3(y)=0\) and \(v_5(y)=0\), we can write \(y=2\left( q_1^{v_{q_1}(y)}q_2^{v_{q_2}(y)}q_3^{v_{q_3}(y)}\right) \), where \(q_1,q_2,q_3\) are distinct primes greater than 6 and \(v_q(y)\ge 1\) for \(q\in \{q_1,q_2,q_3\}\). Since \(\phi (y)=\phi (R(r_1,r_2))\), it follows that \(v_{q_1}(y)=v_{q_2}(y)=v_{q_3}(y)=1\) and hence

$$\begin{aligned} \left( \frac{q_1-1}{2}\right) \left( \frac{q_2-1}{2}\right) \left( \frac{q_3-1}{2}\right) =5^{r_2-1}3^{r_1-1}. \end{aligned}$$

Therefore, for each \(i\in \{1,2,3\}\), we can write \(q_i=2\cdot 3^{a_i}5^{b_i}+1\) such that

$$\begin{aligned}&a_1+a_2+a_3=r_1-1 , \ b_1+b_2+b_3=r_2-1,\\&a_1+b_1\ne 0, \ a_2+b_2\ne 0, \ a_3+b_3\ne 0,\\&a_1,a_2,a_3,b_1,b_2,b_3\ge 0. \end{aligned}$$

Therefore,

$$\begin{aligned} y&=2(2\cdot 3^{a_1}5^{b_1}+1)(2\cdot 3^{a_2}5^{b_2}+1)(2\cdot 3^{a_3}5^{b_3}+1)\nonumber \\&=2\left( 2^33^{r_1-1}5^{r_2-1}+2^2\left( \sum _{i=1}^{3}3^{r_1-a_i-1}5^{r_2-b_i-1}\right) +2\left( \sum _{i=1}^{3}3^{a_i}5^{b_i}\right) +1\right) \nonumber \\&=2\left( 2^33^{r_1-1}5^{r_2-1}+2^23^{r_1-1}5^{r_2-1}\left( \sum _{i=1}^{3}3^{-a_i}5^{-b_i}\right) +2\left( \sum _{i=1}^{3}3^{a_i}5^{b_i}\right) +1\right) . \end{aligned}$$
(4)

Since \(a_i+b_i\ge 1\) for each \(i=1,2,3\), we have

$$\begin{aligned} \sum _{i=1}^{3}3^{-a_i}5^{-b_i}\le \sum _{i=1}^{3}3^{-(a_i+b_i)}\le 1. \end{aligned}$$
(5)

Applying Lemma 35, we have

$$\begin{aligned} \sum _{i=1}^{3}3^{a_i}5^{b_i}\le 3^{r_1-1}5^{r_2-1} . \end{aligned}$$
(6)

Inserting inequalities (5) and (6) in the right-hand side of (4), we get

$$\begin{aligned} y&\le 2\left( 2^33^{r_1-1}5^{r_2-1}+2^23^{r_1-1}5^{r_2-1}+2\cdot 3^{r_1-1}5^{r_2-1}+1\right) \\&\le 2\cdot 3^{r_1-1}5^{r_2-1}(8+4+2+1)= R(r_1,r_2). \end{aligned}$$

\(\square \)

Lemma 38

Suppose \(r_1, r_2, y\in {\mathbb {N}}\) and \(r_2>2\) satisfy \(\phi (y)=\phi (R(r_1,r_2))\), \(|W(y)|=4,\) \(v_2(y)=1,\) \(v_3(y)\ge 1\) and \(v_5(y)=0\). Then \(y\le R(r_1,r_2)\).

Proof

Since \(|W(y)|=4\), \(v_2(y)=1\), \(v_3(y)\ge 1\) and \(v_5(y)=0\), we can write \(y=2\big (3^{v_3(y)}q_1^{v_{q_1}(y)}q_2^{v_{q_2}(y)}\big )\), where \(q_1,q_2\) are distinct primes greater than 6 and \(v_q(y)\ge 1\) for \(q\in \{q_1,q_2,3\}\). Since \(\phi (y)=\phi (R(r_1,r_2))\), it follows that \(v_{q_1}(y)=v_{q_2}(y)=1, v_3(y)\le r_1\), and hence

$$\begin{aligned} \left( \frac{q_1-1}{2}\right) \left( \frac{q_2-1}{2}\right) =5^{r_2-1}3^{r_1-v_3(y)}. \end{aligned}$$

Therefore, we can write \(q_1=2\cdot 3^{a_1}5^{a_2}+1\) and \(q_2=2\cdot 3^{b_1}5^{b_2}+1\) such that \(a_1+b_1=r_1-v_3(y)\), \(a_2+b_2=r_2-1,\ a_1+a_2\ne 0,\ b_1 +b_2\ne 0\) and \(a_1,a_2,b_1,b_2\ge 0\).

Therefore,

$$\begin{aligned} y&=2\cdot 3^{v_3(y)}\big (2\cdot 3^{a_1}5^{a_2}+1\big )\big (2\cdot 3^{b_1}5^{b_2}+1\big ) \\&=2\big (2^2\cdot 3^{a_1+b_1+v_3(y)}5^{a_2+b_2}+2(3^{a_1+v_3(y)}5^{a_2}+ 3^{b_1+v_3(y)}5^{b_2}\big )+3^{v_3(y)}). \end{aligned}$$

Inserting \(a_1+b_1=r_1-v_3(y)\) and \(a_2+b_2=r_2-1\) in the above equation, we get

$$\begin{aligned} y = 2\big (2^2\cdot 3^{r_1}5^{r_2-1}+2\big (3^{r_1-b_1}5^{a_2}+ 3^{r_1-a_1}5^{b_2}\big )+3^{v_3(y)}\big ). \end{aligned}$$
(7)

From Lemma 34, we have

$$\begin{aligned} 5^{a_2}+5^{b_2}\le {\left\{ \begin{array}{ll} 5^{r_2-1} &{}\text{ if } a_2,b_2>0,\\ 1+5^{r_2-1} &{}\text{ else. } \end{array}\right. } \end{aligned}$$
(8)

We are now going to consider the following cases depending on the value of \(a_1\) and \(b_1\).

Case 1. If \(a_1\) and \(b_1\) are positive, then \(a_1+b_1\ge 2\). Using this along with the conditions \(a_1+b_1+v_3(y)=r_1\) and \(v_3(y)\ge 1\), we get \(r_1\ge 3\) and \(v_3(y)\le r_1-2\). Applying these in the right-hand side of equation (7), we have

$$\begin{aligned} y&\le 2\big (2^2\cdot 3^{r_1}5^{r_2-1}+2\cdot 3^{r_1-1}\big (5^{a_2}+ 5^{b_2}\big )+3^{r_1-2}\big ). \end{aligned}$$

Inserting the value of \(5^{a_2}+5^{b_2}\) from (8) in the above inequality, we have

$$\begin{aligned} y&\le 2\cdot 3^{r_1-2}\big (36\cdot 5^{r_2-1}+6\big (1+5^{r_2-1}\big )+1\big )\\&\le 2\cdot 3^{r_1-2}\big (45\cdot 5^{r_2-1}-3\cdot 5^{r_2-1}+7\big )\\&\le 2\cdot 3^{r_1}5^{r_2}=R(r_1,r_2) \ \text { for each } r_2\ge 2. \end{aligned}$$

Case 2. If \(a_1=0\) and \(b_1=0\), then \(v_3(y)=r_1\) due to the fact that \(a_1+b_1=r_1-v_3(y)\). Applying these in equation (7), we have

$$\begin{aligned} y\le 2\cdot 3^{r_1}\big (4\cdot 5^{r_2-1}+2\big (5^{a_2}+ 5^{b_2}\big )+1\big ). \end{aligned}$$

Since \(a_1+a_2\ne 0\) and \(b_1+b_2\ne 0\), it follows that \(a_2,b_2>0\). Hence \(a_2,b_2\le r_2-2\) because \(a_2+b_2=r_2-1\). Using this in the previous inequality gives us

$$\begin{aligned} y&\le 2\cdot 3^{r_1}\big (4\cdot 5^{r_2-1}+4\cdot 5^{r_2-2}+1\big )\\&\le 2\cdot 3^{r_1}\big (5^{r_2}-5^{r_2-2}+1\big )\le R(r_1,r_2), \end{aligned}$$

since \(r_2=1+a_2+b_2\ge 3\).

Case 3. The remaining cases are in which exactly one of \(a_1\) and \(b_1\) is zero. Without loss of generality, assume that \(a_1=0\) and \(b_1\ne 0\).

If \(b_2\ge 1\), we get \(r_2\ge 2\) and \(a_2\le r_2-2\) because \(a_2+b_2=r_2-1\). Since \(a_1=0\) and \(a_1+a_2\ne 0\), we have \(a_2\ge 1\), and hence \(b_2\le r_2-2\). Equation (7) gives

$$\begin{aligned} y&\le 2\big (2^2\cdot 3^{r_1}5^{r_2-1}+2\big (3^{r_1-1}5^{r_2-2}+ 3^{r_1}5^{r_2-2}\big )+3^{r_1}\big )\\&\le 2\cdot 3^{r_1}\big (2^2\cdot 5^{r_2-1}+ 4 \cdot 5^{r_2-2}+1\big ) \\&\le 2\cdot 3^{r_1}\big (2^2\cdot 5^{r_2-1}+ 5^{r_2-1}-5^{r_2-2}+1\big )\\&\le 2\cdot 3^{r_1}\big (5^{r_2}-5^{r_2-2}+1\big )\\&\le 2\cdot 3^{r_1}5^{r_2}, \text { since } r_2\ge 2. \end{aligned}$$

Now, in the case \(b_2=0\), we have \(a_2=r_2-1\). Since \(a_1=0\) and \(a_2+a_1\ne 0\), it follows that \(a_2\ge 1\) and hence \(r_2\ge 2\). Then equation (7) gives

$$\begin{aligned} y&\le 2\big (2^2\cdot 3^{r_1}5^{r_2-1}+2\big (3^{r_1-1}5^{r_2-1}+ 3^{r_1}\big )+3^{r_1}\big )\\&\le 2\big (3^{r_1}5^{r_2}-3^{r_1-1}5^{r_2-1}+3^{r_1+1}\big ) \le R(r_1,r_2),\text { since } r_2 > 2. \end{aligned}$$

Hence \(y\le R(r_1,r_2)\) for \(r_1, r_2 \in {\mathbb {N}},\ r_2>2.\) \(\square \)

PROPOSITION 39

\(R(r_1,r_2)\) lies in \(N_2\) for each \(r_1, r_2 \in {\mathbb {N}},\ r_2>2.\)

Proof

Let y be an even number such that \(\phi (y)=\phi (R(r_1,r_2))\). Since \(v_2(\phi (R(r_1,r_2)))=3\), it follows that \(v_2(\phi (y))=3\). This means that y can have atmost 4 prime factors. If \(|W(y)|\le 3\), then

$$\begin{aligned} D(W(y), W(R(r_1,r_2)))\ge 1, \end{aligned}$$

by Lemma 33. This gives \(y\le R(r_1,r_2)\), by Lemma 32. Now we consider the case \(|W(y)|=4\). Since \(v_2(\phi (y))=3\), it follows in this case that \(v_2(y)=1\).

Suppose \(v_5(y)\ge 1\), then \(v_2(\phi (y))\ge 4\). It follows that \(v_2(\phi (R(r_1,r_2)))\ge 4\) which contradicts the fact that \(v_2(\phi (R(r_1,r_2)))=3\). Therefore, \(v_5(y)=0\).

If \(v_3(y)\ge 1\), Lemma 38 ensures that \(y\le R(r_1,r_2)\). If \(v_3(y)=0\), then Lemma 37 gives \(y\le R(r_1,r_2)\). Therefore \(R(r_1,r_2)\in N_2\) in any case. \(\square \)

Remark 40

From Proposition 2(ii), we get that any element in \(N_1\), all of whose prime factors are less than some prime p, has bounded exponents for its prime factors. But, as seen above from Proposition 39, this is not the case for elements in \(N_2\). In fact, \(R(r_1, r_2) \in N_2\) for \(r_1, r_2 \in {\mathbb {N}},\ r_2>2.\)

So, this raises the following question: For a given odd prime p, do there exist non-negative integers \(d_q\) corresponding to each odd prime \(q < p\) such that \(r_q>d_q\) for each \(q<p \Rightarrow 2\prod _{2<q < p} q^{r_q} \in N_2\)? The numbers \(R(r_1, r_2)\) and \(K_{3,r}\) answer this question in the affirmative for \(p=7\) and \(p=5\) respectively.

Now we are going to give another infinite family of elements in \(N_2\) in which the odd prime factors of elements are Fermat primes.

Lemma 41

If \(\phi (x)=2^r\) for some \(x,r\in {\mathbb {N}},\) then there exist \(b,n\in {\mathbb {N}}\cup \{0\}\) and a sequence of distinct Fermat primes \((F_{i_j})^{j=n}_{j=1}\) such that

$$\begin{aligned} x=2^b\prod _{j=1}^{n}F_{i_j}. \end{aligned}$$

Proof

We observe that if \(\phi (x)=2^r\) and if an odd prime \(q \mid x\), then \((q-1) \mid 2^r\) which implies that q is of the form \(2^l + 1\) for some \(l \in {\mathbb {N}}\). But it is well-known that if \(2^l + 1\) is a prime, then \(l=2^{\alpha }\) for some \(\alpha \ge 0\) (see [8, Theorem 17]). Hence, \(q=2^{2^{\alpha }}+1\), a Fermat prime. Also, \(v_{q}(x)=1\) for each such \(q \mid x\). If not, then \(q \mid \phi (x)=2^r\), a contradiction since q is odd. Therefore, x will be of the form \(x=2^b\prod _{j=1}^{n}F_{i_j}\), where \(i_j\in {\mathbb {N}}, \ b, n\in {\mathbb {N}}\cup \{0\}.\) \(\square \)

PROPOSITION 42

Let \(F_j\) denote the j-th Fermat prime for \(j \in {\mathbb {N}}\). Suppose \(F_1, F_2, \dots , F_k\) exist. If \(F_{k + 1}\) also exists,  then \(2^aF\in N_2,\) where \(F=\prod _{i=1}^{k}F_i\) and \(1\le a \le \log _2(F_{k+1}-1)\). If \(F_{k+1}\) does not exist,  then \(2^aF\in N_2\) for each \(a\in {\mathbb {N}}.\)

Proof

Define \(y:=2^a\prod _{i=1}^{k}F_i\) with \(a\in {\mathbb {N}}\). To prove \(y\in N_2\), it is enough to show that if x is any even integer satisfying \(\phi (y)=\phi (x)\), then \(x\le y\). This can be observed using the fact that elements of \(N_2\) are even.

Let x be an even integer satisfying \(\phi (x)=\phi (y)\). Since \(\phi (y)= 2^r\) for some \(r\in {\mathbb {N}}\), it follows that \(\phi (x)=2^r\) for some \(r\in {\mathbb {N}}\). Then Lemma 41 gives \(x=2^b\prod _{j=1}^{n}F_{i_j}\) for some \(b \in {\mathbb {N}}\) and \(n\in {\mathbb {N}}\cup \{0\}\). Since \(\phi (x)=\phi (y)\), we have

$$\begin{aligned} a=b+\sum _{j=1}^{n}\log _2(F_{i_j}-1)-\sum _{j=1}^{k}\log _2(F_j-1). \end{aligned}$$
(9)

If \(F_{k+1}\) exists, then

$$\begin{aligned} |W(x)|> |W(y)|&\Rightarrow \sum _{j=1}^{n}\log _2(F_{i_j}-1)-\sum _{j=1}^{k}\log _2(F_j-1)\ge \log _2(F_{k+1}-1)\\&\Rightarrow a\ge b+ \log _2(F_{k+1}-1)\ \text { by equation } (9) \\&\Rightarrow a\ge 1+ \log _2(F_{k+1}-1)\ \text { since } b\in {\mathbb {N}}. \end{aligned}$$

Therefore,

$$\begin{aligned} a\le \log _2(F_{k+1}-1)&\Rightarrow |W(x)|\le |W(y)|\\&\Rightarrow D(W(x), W(y))\ge 1 \ \text { by using Lemma }~33\\&\Rightarrow x\le y\ \text { by Lemma }~32. \end{aligned}$$

On the other hand, if \(F_{k+1}\) does not exist, then \(W(x)\subset W(y)\) for each \(a\in {\mathbb {N}}\). It follows that \(D(W(x), W(y))\ge 1\), using Lemma 33. Then Lemma 32 gives \(x\le y\). \(\square \)

Only five Fermat primes are known till date. From the above proposition, one can see that there exist elements in \(N_2\setminus N_1\) which are divisible by arbitrarily large powers of 2. In all the earlier results, the elements obtained were divisible by 2 but not by 4.

COROLLARY 43

For a positive integer r,  there exist infinitely many integers l such that \(l \equiv 0 \pmod {2^r}\) and \(l \in N_2\setminus N_1.\)

DEFINITION 44

Let \(F_j\) denote the j-th Fermat prime for \(j \in {\mathbb {N}}\) and let \(H=\left\{ k \in {\mathbb {N}}:F_k \text { exists}\right\} \). Define

By collecting Propositions 30, 39 and 42 , we get that (i) \({\mathcal {K}}_{\max }, {\mathcal {R}}\) and \({\mathcal {F}}\) are infinite subsets of \(N_2\) and (ii) \({\mathcal {K}}_{\min }\) is an infinite subset of \(N_3\) in which infinitely many elements are composite. Proposition 2(ii) gives that only finitely many elements of \({\mathcal {K}}_{\max }, {\mathcal {R}}\) and \({\mathcal {F}}\) belong to \(N_1\).

Theorem 5

\({\mathcal {K}}_{\max }, {\mathcal {R}}\) and \({\mathcal {F}}\) are infinite subsets of \(N_2\) in which only finitely many elements are in \(N_1\). \({\mathcal {K}}_{\min }\) is an infinite subset of \(N_3\) in which infinitely many elements are composite.

3.2 Proof of Theorem 6

In the previous results, we looked at several families of elements in \(N_2\) and \(N_3\). Now, we would like to compare the values of \(N_2(m)\) and \(N_3(m)\). In the following proposition, we are going to give upper and lower bounds for \(N_2(m)\) and \(N_3(m)\) and we also look at the ratio \(N_2(m)/N_3(m)\).

Lemma 45

Let m be an odd integer. If u is an odd integer satisfying \(\phi (u)=4m,\) then

$$\begin{aligned} u=\displaystyle \frac{(2z_1+1)(2z_2+1)}{z_1z_2}m~\text { or }~ \displaystyle \frac{(4z_3+1)}{z_3}m, \end{aligned}$$

where \(z_1z_2{\mid } m,\) \(z_3{\mid } m,\) \(z_1\ne z_2\) and  2z\(_1\) + 1, 2z\(_2\) + 1, 4z\(_3\) + 1 are primes. Also,  \(4m<u\le 7m\).

Proof

Any odd integer u satisfying \(\phi (u)=4m\) can have at most two prime factors. If u has two distinct prime factors \(q_1\) and \(q_2\) such that \(q_1<q_2\) and \(q_1, q_2 \equiv 3 \pmod {4}\) (since \(v_2(\phi (u))=2\)), then

$$\begin{aligned} u\left( \frac{(q_1-1)(q_2-1)}{q_1q_2}\right) =4m, \text { i.e., } u=\frac{4q_1q_2}{(q_1-1)(q_2-1)}m . \end{aligned}$$

Since \(u,q_1,q_2\) and m are all odd, we have \(q_1=2z_1+1, \ q_2=2z_2+1\) for some odd integers \(z_1, z_2\) with \(z_1 < z_2\). Moreover, \(q_1q_2|u \Rightarrow (q_1-1)(q_2-1) \mid \phi (u)=4m\), i.e., \(z_1z_2 \mid m\). Using this in the value of u, we have

$$\begin{aligned} u=\displaystyle \frac{(2z_1+1)(2z_2+1)}{z_1z_2}m, \end{aligned}$$

where \(z_1z_2\) divides m. Clearly, \(u>4m\) and it can take a maximum value of 7m if \(z_1 = 1, z_2 = 3\). Therefore, \(u\le 7m\). If u has only one prime factor q with \(q \equiv 5 \pmod {8}\) (since \(v_2(\phi (u))=2\)), then

$$\begin{aligned} u=\frac{4qm}{q-1}. \end{aligned}$$

Now, \(q \equiv 5 \pmod {8}\), and so \(q=4z_3+1\) for some odd integer \(z_3\). Therefore,

$$\begin{aligned} u=\frac{(4z_3+1)}{z_3}m. \end{aligned}$$

Note that \(z_3,4z_3 + 1\) are co-prime integers and hence \(z_3 \mid m\). Clearly \(u>4m\) and it can take a maximum value of 5m if \(z_3 = 1.\) \(\square \)

Now we proceed to prove Theorem 6(i) and 6 (ii).

Proof of Theorem 6(i)

If \(m\equiv 2\pmod {4}\) and \(\phi (x)=m\) is solvable, then by Lemma 28, \(m=q^{r}(q-1)\) for some \(q\equiv 3 \pmod {4}\) . Since \(q^r(q-1)+1\le q^{r+1}\le \frac{3m}{2}\), we have \(m<N_3(m)\le \frac{3m}{2}\) and \(2m<N_2(m) \le 3m\), by Proposition 30. If \(m \equiv 4 \pmod {8}\), firstly, note that the proposition is true for \(m=4,\) since \(N_3(4)=5\) and \(N_2(4)=12\). So, we can assume that \(m \ge 12\). If \(\phi (x)=m\) for \(m \equiv 4 \pmod {8}\), then \(v_2(x)\le 2\).

Suppose that there exists an integer z such that \(v_2(z)=2\) and \(\phi (z)=m\), where \(m=4m_0, \ m_0\) being an odd integer. If \(z=4y\), then y is an odd integer satisfying \(\phi (x)=2m_0\). Then by Lemma 28, \(y=p^{\alpha }\) for some \(\alpha \ge 1\) and prime \(p \equiv 3\pmod {4}\). Therefore, \(z=4p^{\alpha }\). If \(p>3\), then \(3p^{\alpha }<z<6p^{\alpha }\) and \(\phi (3p^{\alpha })=\phi (z)= \phi (6p^{\alpha })\). If \(p=3\) and \(\alpha >1\), then \( 7\times 3^{\alpha -1}<z< 14 \times 3^{\alpha -1}\) and again they have the same \(\phi \) value. Hence, if \(\phi (z)=m, \ m \equiv 4 \pmod {8}\) and \(v_2(z)=2\), then \(z \notin \{N_2(m), N_3(m)\}\). Moreover, an odd integer \(l \in \phi ^{-1}(m) \text { iff } 2l \in \phi ^{-1}(m)\). Therefore, \(v_2(N_3(m))=0\) and \(v_2(N_2(m))=1\). Now, note that \(N_3(m)\) and \(\frac{N_2(m)}{2}\) are odd integers satisfying \(\phi (x)=m=4m_1\), where \(m_1\) is odd. Therefore, by Lemma 45, we have \(m<N_3(m)\) and \( \frac{N_2(m)}{2} \le \frac{7m}{4}\) and the result follows. \(\square \)

Proof of Theorem 6(ii)

By Lemma 28, if \(m \in V\), then \(m \equiv 2 \pmod {4} \iff m=q^r(q-1)\) for some prime \(q \equiv 3 \pmod {4}, \ r \ge 0\). Now, by Proposition 30, if \(q^r(q-1)+1\) is composite, then

$$\begin{aligned} \frac{N_2(q^r(q-1))}{N_3(q^r(q-1))}=2. \end{aligned}$$

Else, if \(q^r(q-1)+1\) is prime, then

$$\begin{aligned} \frac{N_2(q^r(q-1))}{N_3(q^r(q-1))}=\frac{2q^{r+1}}{q^r(q-1)+1}=\frac{2q}{q-1+\frac{1}{q^r}}. \end{aligned}$$

It is readily seen that the rightmost quantity lies between 2 and 3 since \(r \ge 0, \ q \ge 3\). \(\square \)

To prove Theorem 6(iii), we give examples of infinite length geometric progressions in \(N_2\) and \(N_3\). For each prime \(q \equiv 3 \pmod {4}\), note that \(\{K_{q, r}\}_{r \in {\mathbb {N}}}\) is a geometric progression in \(N_2\) with common ratio q. Also, we see that \(\{R(r_0, r)\}_{r=3}^{\infty }\) is a geometric progression in \(N_2\) with common ratio 5.

Now, we turn our attention to geometric progressions in \(N_3\). We construct an infinite geometric progression in \(N_3\) with the help of the following lemma.

Lemma 46

Let q be a prime satisfying \(q \equiv 3 \text { or } 7 \pmod {20}\). Then the set \(\{r :q^{r}(q-1) + 1 \text { is composite}\}\) contains an infinite arithmetic progression.

Proof

Suppose that \(q \equiv 3 \pmod {20}\). We observe that \(q^{r}(q-1) + 1\) is divisible by 5 for \(r \equiv 3 \pmod {4}\). Now, if \(q \equiv 7 \pmod {20}\), we see that \(q^{r}(q-1) + 1\) is divisible by 5 for \(r \equiv 2 \pmod {4}\). So, in any case, the set \(\{r :q^{r}(q-1) + 1 \text { is composite}\}\) contains an infinite arithmetic progression. \(\square \)

If \(q \equiv 3 \text { or } 7 \pmod {20}\), then \(k_{q, r} \in N_3\) for each \(r \in {\mathbb {N}}\). As the set \(S=\{r :q^{r}(q-1) + 1 \text { is composite}\}\) contains an infinite arithmetic progression, the set \(\{k_{q, r} :r \in S\}\) contains an infinite geometric progression. So, corresponding to each such q, there is an infinite geometric progression. This implies an infinite family of such geometric progressions in \(N_3\) due to the following result of Dirichlet [8, Theorem 15, page 16].

PROPOSITION 47

(Dirichlet’s theorem)

Suppose \((a, q) = 1\). Then there are infinitely many primes p satisfying \(p \equiv a \pmod {q}.\)

4 Arithmetic progressions in sparse sets

Szemerédi’s theorem assures the existence of arbitrarily long arithmetic progressions in a set having positive asymptotic density. But the converse is not necessarily true. For example, the set of prime numbers has zero asymptotic density but contains arbitrarily long arithmetic progressions, as proved by Green and Tao [6].

DEFINITION 48

Let \(A\subset {\mathbb {P}}\). Then \(Rd(A)=\limsup _{N\rightarrow \infty }\frac{|A\cap [1, N]|}{\pi (N)}\) defines the relative upper density of A with respect to \({\mathbb {P}}\), where \(\pi (N)\) denotes the number of primes less than or equal to N.

PROPOSITION 49

[6]

Let A be any subset of the prime numbers of positive relative upper density Rd(A). Then A contains infinitely many arithmetic progressions of length k for all k.

Let \(f:V \rightarrow {\mathbb {N}}\) be such that \(f(m)\in \phi ^{-1}(m)\). By Corollary 22, \({{\bar{d}}}(f(V))=0\). We observe that it satisfies the hypothesis of the following famous conjecture of Erdős and Turán.

Conjecture 1

(Erdős and Turán). If \(A\subset {\mathbb {N}}\) is such that \(\sum _{n\in A}n^{-1}\) diverges, then A contains arbitrarily long arithmetic progressions.

PROPOSITION 50

Let \(f:V \rightarrow {\mathbb {N}}\) be such that \(f(m)\in \phi ^{-1}(m)\). Then \(\sum _{m\in V}\frac{1}{f(m)}\) diverges.

Proof

We have

$$\begin{aligned} \displaystyle \sum _{n\in N_2}\frac{1}{n}\le \displaystyle \sum _{m\in V}\frac{1}{f(m)}\le \displaystyle \sum _{n\in N_3}\frac{1}{n}. \end{aligned}$$

If \(m\equiv 2\pmod {4}\), then \(N_2(m) \le 3N_3(m)\), by Proposition  6(ii). So, it follows that

$$\begin{aligned} \displaystyle \sum _{m\in V}\frac{1}{N_2(m)} \ge \displaystyle \sum _{\begin{array}{c} m\equiv 2 \pmod {4} \\ m \in V \end{array}}\frac{1}{3N_3(m)} \ge \displaystyle \sum _{p\equiv 3\pmod {4}}\frac{1}{3p}, \end{aligned}$$

since \(N_3\) contains each odd prime. The right most series is divergent (see [2, Chapter 4]) and the result follows. \(\square \)

Hence we may expect arbitrarily long arithmetic progressions in f(V). Indeed, the following result due to Erdős [3, p. 15] and Proposition 49 confirm our intuition.

PROPOSITION 51

[3]

Suppose \(m \in V\) with \(|\phi ^{-1}(m)|=k\) for \(k\ge 2\). Then there exists a set \(P\subset {\mathbb {P}}\) such that \(Rd(P) > 0\) and for each \(p\in P,\) \(\phi ^{-1}(m(p-1))=p\phi ^{-1}(m)\).

Proof of Theorem 7

Consider the set \(V_1=\{m\in V:|\phi ^{-1}(m)|=2\}\) and let \(m'\in V_1\). Then, by Proposition 51, there exists \(P_1\subset {\mathbb {P}}\) such that \(Rd(P_1)>0\) and \(m'(p-1)\in V_1\) for each \(p\in P_1\). Now, consider the sets \(P_2=\{p\in P_1:pN_2(m')\in f(V)\}\) and \(P_3=\{p\in P_1:pN_3(m')\in f(V)\}\). By the definition of the set f(V), we have \(P_2\cap P_3=\varnothing \) and \(P_2\cup P_3=P_1\). Therefore, at least one of the sets \(P_2\) or \(P_3\) has positive relative density in \({\mathbb {P}}\) and thus, by Proposition 49, contains arbitrarily long arithmetic progressions. Therefore, one of the subsets \(N_2(m')P_2\) or \(N_2(m')P_3\) of f(V) contains arbitrarily long arithmetic progressions and the result follows. \(\square \)

5 Questions

As discussed in Remark 40, we raise the following question about elements in \(N_2\).

Question 1

For a given odd prime p, do there exist non-negative integers \(d_q\) for each prime \(q < p\) such that \( r_q>d_q\) for each prime \(q<p \Rightarrow 2\prod _{2<q < p} q^{r_q} \in N_2\)?

We have observed that the Banach densities of V and \(N_1\) are zero. Also, the asymptotic density \({{\bar{d}}}(N_2)\) of \(N_2\) is zero. Since \(N_1 \subset N_2,\ d^{*}(N_1)=0\), and there is a bijection \(f_{\max }:V \rightarrow N_2\) given by \(f_{\max }(m)=N_2(m)\), it may seem that \(d^{*}(N_2)=0\). But, on the other hand, consider the function \(f :{\mathbb {N}}^2 \rightarrow {\mathbb {N}}\) (where \({\mathbb {N}}^2\) is the set of square numbers \(\{1, 4, 9, 16, \dots \}\)) defined by \(f(y)=\sqrt{y}\), \(\sqrt{y}\) being the unique positive square root of y. By Theorem 4(a), we see that \(d^{*}({\mathbb {N}}^2)=0\). So f is a bijection from a zero Banach density set to a set with positive Banach density. Now, the function \(f_{\max }\) is not increasing and hence, it suggests that \(N_2\) may have a positive Banach density. So, it is interesting to ask the following question.

Question 2

What is the Banach density of \(N_2\) and \(N_3\)?

In fact, except for Følner sequences of type \((a_n, a_n(1 + \alpha _n)]\), where \(\alpha _n \rightarrow 0,\ a_n \rightarrow \infty \), and \(\alpha _n a_n \rightarrow \infty \), one can see that the upper density with respect to other Følner sequnces is zero.